NURS 8310C: Epidemiology and Population Health | Week 9

NURS 8310C: Epidemiology and Population Health | Week 9

As you have examined in this course, the scope of epidemiology has broadened over the years to include the global investigation of chronic, environmental, and genetic diseases and other health-related conditions. Yet, the practice of epidemiology has its roots in the study of infectious disease, global epidemics, and pandemics. Perhaps you recall the widely reported concerns about the H1N1 virus, SARS, or the reemergence of measles, tuberculosis, or whooping cough. The emergence and reemergence of infectious disease has long held the attention of epidemiologists, as well as the general public. NURS 8310C: Epidemiology and Population Health | Week 9

This week, you will explore the investigative process epidemiologists use to examine infectious diseases. You will also submit Assignment 4.

ORDER NOW FOR CUSTOMIZED SOLUTION PAPERS

Learning Objectives – NURS 8310/ NURS 8310F/ NURS 8310M/ NURS 8310A/ NURS 8310C: Epidemiology and Population Health | Week 9

Students will:

  • Analyze the investigative process for disease outbreaks
  • Evaluate the application of health care interventions on emerging or reemerging infectious diseases
  • Formulate an evaluation plan for a population health intervention

Learning Resources

Required Readings

Friis, R. H., & Sellers, T. A. (2014). Epidemiology for public health practice (5th ed.). Sudbury, MA: Jones & Bartlett.

    • Chapter 12, “Epidemiology of Infectious Diseases”

In this chapter, the authors examine the epidemiology of infectious diseases, one of the most familiar applications of epidemiology.

Centers for Disease Control and Prevention. (2011). CDC says “Take 3” actions to fight the flu. Retrieved from http://www.cdc.gov/flu/protect/preventing.htm

This page contains the CDC’s most up-to-date recommendations regarding the prevention of seasonal flu. In addition to this page, you may wish to explore the CDC’s Seasonal Influenza home page, http://www.cdc.gov/flu/

World Health Organization. (2012). Disease outbreak news. Retrieved from http://www.who.int/csr/don/en/

The World Health Organization (WHO) provides information on the most recent disease outbreaks around the world. Stay up to date by visiting this site.

HealthMap. (2007). Retrieved from http://www.healthmap.org/en

Explore this interactive map that lists disease outbreaks around the world.

Centers for Disease Control and Prevention. (2011). Morbidity and mortality weekly report: Summary of notifiable diseases. Retrieved from http://www.cdc.gov/mmwr/mmwr_nd/index.html

Review the most current report on infectious diseases as reported by health care providers to state or local authorities. According to the CDC, “A disease is designated as notifiable if timely information about individual cases is considered necessary for prevention and control of the disease.” This report highlights infectious diseases reported in 2009. NURS 8310C: Epidemiology and Population Health | Week 9

Required Media

Laureate Education (Producer). (2012). Epidemiology and population health: Infectious disease: Two case studies [Video file]. Baltimore, MD: Author.

Note: The approximate length of this media piece is 8 minutes.

In this week’s program, the presenters discuss HIV and AIDS.

Optional Resources

Ghosh, T. S., Patnaik, J. L., Alden, N. B., & Vogt, R. L. (2008). Internet-versus telephone-based local outbreak investigations. Emerging Infectious Diseases, 14(6), 975–977.

Seto, E. Y.W., Soller, J. A. & Colford, J. M. Jr. (2007). Strategies to reduce person-to-person transmission during widespread Escherichia coli O157:H7 outbreak. Emerging Infectious Diseases, 13(6), 860–866.

Discussion: Investigating Pandemics and Epidemics

Some of the most notable epidemics include the bubonic plague in the 14th century, smallpox in the 18th century, and influenza in the 20th century. Reportedly, the bubonic plague caused over 137 million deaths, whereas the death toll associated with influenza was 25 million (Ernst, 2001). These are dramatic examples of the kinds of acute outbreaks that led to the practice of epidemiology.

Many epidemiologists and health care professionals are concerned about the next potential pandemic or epidemic. With the increased mobility of society, the spread of infectious diseases continues to pose a serious threat. For this Discussion, you will investigate pandemics and epidemics using epidemiological tools, and you will consider strategies for mitigating disease outbreaks.

To prepare:

  • Using the Learning Resources, consider examples of emerging or reemerging infectious diseases that are occurring locally, nationally, or abroad. Then, select one example on which to focus.
  • Explore the epidemiological investigative process used to identify the emerging or reemerging infectious disease or outbreak.
  • Examine your selected infectious disease using the epidemiologic triangle and vector theory.
  • Consider how health care interventions may reduce the emergence or reemergence of infectious diseases.

By Day 3

Post a cohesive response that addresses the following:

  • Identify the emerging or reemerging infectious disease you selected.
  • Discuss the investigative process used to identify the outbreak, and describe its effect using descriptive epidemiology (person, place, and time).
  • Apply the epidemiologic triangle and vector theory to your selected outbreak.
  • Evaluate how prior health care interventions, or lack thereof, created the conditions that allowed this infectious disease to emerge.
  • Discuss how the disease outbreak might have been avoided or mitigated. Include agencies, organizations, and resources that could have supported these efforts. If appropriate, consider ongoing efforts to control the outbreak. NURS 8310C: Epidemiology and Population Health | Week 9

Read a selection of your colleagues’ responses.

By Day 6

Respond to at least two of your colleagues in one or more of the following ways:

  • Ask a probing question, substantiated with additional background information, evidence, or research.
  • Share an insight from having read your colleagues’ postings, synthesizing the information to provide new perspectives.
  • Offer and support an alternative perspective using readings from the classroom or from your own research in the Walden Library.
  • Validate an idea with your own experience and additional research.
  • Make a suggestion based on additional evidence drawn from readings or after synthesizing multiple postings.
  • Expand on your colleagues’ postings by providing additional insights or contrasting perspectives based on readings and evidence. NURS 8310/ NURS 8310F/ NURS 8310M/ NURS 8310A/ NURS 8310C: Epidemiology and Population Health | Week 9

NURS 6053: Module 4: Communication And Relationship Building – Weeks 7-9

NURS 6053: Module 4: Communication And Relationship Building – Weeks 7-9

Students will:

  • Assess work environments for workplace civility
  • Analyze strategies to address workplace incivility
  • Analyze evidence-based theories for promoting organizational health
  • Recommend strategies for improving workplace environments
Due By Assignment
Week 7, Days 1–2 Read the Learning Resources.
Compose your initial Discussion post.
Week 7, Day 3 Post your initial Discussion post.
Begin to compose your Assignment.
Week 7, Days 4-5 Review peer Discussion posts.
Compose your peer Discussion responses.
Continue to compose your Assignment.
Week 7, Day 6 Post two peer Discussion responses.
Week 7, Day 7 Wrap up Discussion.
Week 8, Days 1-7 Continue to compose your Assignment.
Week 9, Days 1-6 Continue to compose your Assignment.
Week 9, Day 7 Deadline to submit your Assignment.

Permalink: https://eaziessay.com/nurs-6053-module…ilding-weeks-7-9/

Learning Resources – NURS 6053: Module 4: Communication and Relationship Building – Weeks 7-9

Note: To access this week’s required library resources, please click on the link to the Course Readings List, found in the Course Materials section of your Syllabus.

Required Readings

Marshall, E., & Broome, M. (2017). Transformational leadership in nursing: From expert clinician to influential leader (2nd ed.). New York, NY: Springer.

  • Chapter 5, “Collaborative Leadership Contexts: Networks, Communication, Decision Making, and Motivation” (pp. 121–144)
  • Chapter 9, “Creating and Shaping the Organizational Environment and Culture to Support Practice Excellence” (pp. 247–278)
  • Chapter 10, “Building Cohesive and Effective Teams” (pp. 279–298) NURS 6053: Module 4: Communication And Relationship Building – Weeks 7-9

    ORDER NOW FOR CUSTOMIZED SOLUTION PAPERS

Select at least ONE of the following:

Clark, C. M., Olender, L., Cardoni, C., & Kenski, D. (2011). Fostering civility in nursing education and practice: Nurse leader perspectives. Journal of Nursing Administration, 41(7/8), 324–330. doi:10.1097/NNA.0b013e31822509c4

Note: You will access this article from the Walden Library databases.

Clark, C. M. (2018). Combining cognitive rehearsal, simulation, and evidence-based scripting to address incivility. Nurse Educator. doi:10.1097/NNE.0000000000000563

Note: You will access this article from the Walden Library databases.

Clark, C. M. (2015). Conversations to inspire and promote a more civil workplace. American Nurse Today, 10(11), 18–23. Retrieved from https://www.americannursetoday.com/wp-content/uploads/2015/11/ant11-CE-Civility-1023.pdf

Griffin, M., & Clark, C. M. (2014). Revisiting cognitive rehearsal as an intervention against incivility and lateral violence in nursing: 10 years later. Journal of Continuing Education in Nursing, 45(12), 535–542. doi:10.3928/00220124-20141122-02

Note: You will access this article from the Walden Library databases.

Document: Work Environment Assessment Template (Word document)

Required Media

TEDx. (2017, April). Jody Hoffer Gittell: The power of a simple idea [Video file]. Retrieved from https://www.youtube.com/watch?v=X7nL5RC5kdE

Laureate Education (Producer). (2009a). Working with Groups and Teams [Video file]. Baltimore, MD: Author.

Discussion: Workplace Environment Assessment

How healthy is your workplace?

You may think your current organization operates seamlessly, or you may feel it has many issues. You may experience or even observe things that give you pause. Yet, much as you wouldn’t try to determine the health of a patient through mere observation, you should not attempt to gauge the health of your work environment based on observation and opinion. Often, there are issues you perceive as problems that others do not; similarly, issues may run much deeper than leadership recognizes. NURS 6053: Module 4: Communication And Relationship Building – Weeks 7-9

There are many factors and measures that may impact organizational health. Among these is civility. While an organization can institute policies designed to promote such things as civility, how can it be sure these are managed effectively? In this Discussion, you will examine the use of tools in measuring workplace civility.

To Prepare:

  • Review the Resources and examine the Clark Healthy Workplace Inventory, found on page 20 of Clark (2015).
  • Review and complete the Work Environment Assessment Template in the Resources.

By Day 3 of Week 7

Post a brief description of the results of your Work Environment Assessment. Based on the results, how civil is your workplace? Explain why your workplace is or is not civil. Then, describe a situation where you have experienced incivility in the workplace. How was this addressed? Be specific and provide examples.

By Day 6 of Week 7

Respond to at least two of your colleagues on two different days by sharing ideas for how shortcomings discovered in their evaluations and/or their examples of incivility could have been managed more effectively. NURS 6053: Module 4: Communication and Relationship Building – Weeks 7-9. NURS 6053: Module 4: Communication And Relationship Building – Weeks 7-9

NURS 6512 Module 1: Comprehensive Health History

NURS 6512 Module 1: Comprehensive Health History

What’s Happening in This Module?

This course is composed of four (4) separate modules. Each module consists of an overarching topic in which each week within the module includes specific subtopics for learning. As you work through each module, you will have an opportunity to draw upon the knowledge you gain in various Digital Clinical Experiences (DCE) and lab assignment components that will be due throughout each of the modules.

ORDER NOW FOR CUSTOMIZED SOLUTION PAPERS

Module 1: Comprehensive Health History is a 1-week module, Week 1 of the course, in which you will examine how social determinants of health such as age, gender, ethnicity, and environmental situations impact the health and risk assessment of the patients you serve. You will also consider how social determinants of health influence your interview and communication techniques as you work in partnership with a patient to gather data to build an accurate health history. NURS 6512 Module 1: Comprehensive Health History

What do I have to do? When do I have to do it?
Review your Learning Resources Days 1–7, Week 1
Discussion: Building a Comprehensive Health History Post by Day 3 of Week 1, and respond to your colleagues by Day 6 of Week 1.
What’s Coming Up in Module 2: Looking Ahead Review the “Looking Ahead” section for this week. You are encouraged to further review the requirements for the Shadow Health registration process for your digital clinical experiences.

Go to the Week’s Content

Week 1: Building a Comprehensive Health History

According to a 2011 Gallup poll, nurses are ranked as the most trusted professionals in the United States. One of the most admired nursing skills is the ability to put patients at ease. When patients enter into a healthcare setting, they are often apprehensive about sharing personal health information. Caring nurses can alleviate the hesitance of patients and encourage them to be forthcoming with this information.

The initial health history interview can be an excellent opportunity to develop supportive relationships between patients and nurses. Nurses may employ a variety of communication skills and interview techniques to foster strong bonds with patients and to effectively facilitate the diagnostic process. In conducting interviews, advanced practice nurses must also take into account a range of patient-specific factors that may impact the questions they ask, how they ask those questions, and their complete assessment of the patient’s health.

This week, you will consider how social determinants of health such as age, gender, ethnicity, and environmental situation impact the health and risk assessment of the patients you serve. You will also consider how social determinants of health influence your interview and communication techniques as you work in partnership with a patient to gather data to build an accurate health history. NURS 6512 Module 1: Comprehensive Health History

Learning Objectives

Students will:

  • Analyze communication techniques used to obtain patients’ health histories based upon social determinants of health
  • Analyze health-related risk
  • Apply concepts, theories, and principles related to patient interviewing, diagnostic reasoning, and recording patient information

Learning Resources

Required Readings (click to expand/reduce)
Required Media (click to expand/reduce)

Discussion: Building a Health History

Effective communication is vital to constructing an accurate and detailed patient history. A patient’s health or illness is influenced by many factors, including age, gender, ethnicity, and environmental setting. As an advanced practice nurse, you must be aware of these factors and tailor your communication techniques accordingly. Doing so will not only help you establish rapport with your patients, but it will also enable you to more effectively gather the information needed to assess your patients’ health risks.

For this Discussion, you will take on the role of a clinician who is building a health history for a particular new patient assigned by your Instructor.

Photo Credit: Getty Images/Caiaimage

To prepare:

With the information presented in Chapter 1 of Ball et al. in mind, consider the following:

  • By Day 1 of this week, you will be assigned a new patient profile by your Instructor for this Discussion. Note: Please see the “Course Announcements” section of the classroom for your new patient profile assignment.
  • How would your communication and interview techniques for building a health history differ with each patient?
  • How might you target your questions for building a health history based on the patient’s social determinants of health?
  • What risk assessment instruments would be appropriate to use with each patient, or what questions would you ask each patient to assess his or her health risks?
  • Identify any potential health-related risks based upon the patient’s age, gender, ethnicity, or environmental setting that should be taken into consideration.
  • Select one of the risk assessment instruments presented in Chapter 1 or Chapter 5 of the Seidel’s Guide to Physical Examination text, or another tool with which you are familiar, related to your selected patient.
  • Develop at least five targeted questions you would ask your selected patient to assess his or her health risks and begin building a health history. NURS 6512 Module 1: Comprehensive Health History
By Day 3 of Week 1

Post a summary of the interview and a description of the communication techniques you would use with your assigned patient. Explain why you would use these techniques. Identify the risk assessment instrument you selected, and justify why it would be applicable to the selected patient. Provide at least five targeted questions you would ask the patient.

Note: For this Discussion, you are required to complete your initial post before you will be able to view and respond to your colleagues’ postings. Begin by clicking on the “Post to Discussion Question” link, and then select “Create Thread” to complete your initial post. Remember, once you click on Submit, you cannot delete or edit your own posts, and you cannot post anonymously. Please check your post carefully before clicking on Submit!

Read a selection of your colleagues’ responses.

By Day 6 of Week 1

Respond to at least two of your colleagues on 2 different days who selected a different patient than you, using one or more of the following approaches:

  • Share additional interview and communication techniques that could be effective with your colleague’s selected patient.
  • Suggest additional health-related risks that might be considered.
  • Validate an idea with your own experience and additional research.

Submission and Grading Information

Grading Criteria

To access your rubric:

Week 1 Discussion Rubric

Post by Day 3 of Week 1 and Respond by Day 6 of Week 1

To Participate in this Discussion:

Week 1 Discussion


What’s Coming Up in Module 2?

Photo Credit: Getty Images/iStockphoto

In Module 2, you explore the impact of functional assessments, diversity, and sensitivity in conducting health assessments. You also examine various assessment tools and diagnostic tests used to gather information about patients’ conditions and examine their validity, reliability, and impact in conducting health assessments.

Next week, you will specifically examine functional assessments as they relate to diversity and sensitivity

Registration for Shadow Health

Throughout this course, you will participate in digital clinical experiences using the online simulation tool Shadow Health. The Shadow Health digital clinical experience provides a dynamic, immersive experience designed to improve nursing skills and clinical reasoning through the examination of digital standardized patients. Using Shadow Health you will participate in health histories, focused exams, and a comprehensive assessment.

There will be four Shadow Health assessment components that you will need to complete in Module’s 2 and 3:

  • Health History Assessment (Week 3 & 4)
  • Focused Exam: Cough (Week 5) for a pediatric patient presenting with cough
  • Focused Exam: Chest Pain (Week 7) for an adult patient presenting with chest pain
  • Comprehensive (Head-to-Toe) Physical Assessment (Week 9)

Before you can participate in these simulations, you will need to register for a Shadow Health account. To do this:

  • Go to the Walden Bookstore and purchase access to Shadow Health and the required texts.
  • Once Shadow Health has been purchased, an access code will be emailed to you from the bookstore.
  • Review this video explaining how to register in Shadow Health: https://vimeo.com/275921826/c12d50ee6e
  • Use the Shadow Health link located in the navigation menu on the left in the Blackboard course.
  • Follow the prompts to register in Shadow Health. You will need the access code provided from the bookstore to register. Once registered, Shadow Health should always be accessed via the link in Blackboard.
  • Use only Google Chrome when accessing Shadow Health and make sure all other programs are turned off on your computer. Other browsers do not work well and will not allow the Shadow Health speech to text function to work.
  •  Once registered, complete the Shadow Health Orientation in the Shadow Health website/program and review the videos designed to assist with navigating and completing assignments.
  • Read the Shadow Health Nursing Documentation Tutorial located in the Week 1 Learning Resources.

Note: As nurses you typically use the word assessment to mean completing the physical exam. However, in the SOAP Note format, assessment means diagnosis so start getting in the habit of calling the physical exam exactly that.

Week 2 Case Studies

In Week 2, your Instructor will assign you a case study related to your Discussion by Day 1 of the week. Please make sure to review the “Course Announcements” area of the course to verify your assigned case study. Please plan ahead to ensure you have time to review your case study and your Learning Resources so that you can complete your Discussions and Assignments on time.

Photo Credit: Getty Images/iStockphoto

Next Module

NURS 6512 Assignment 1: Assessment Tools and Diagnostic Tests in Adults and Children Case Study

NURS 6512 Assignment 1: Assessment Tools and Diagnostic Tests in Adults and Children Case Study

When seeking to identify a patient’s health condition, advanced practice nurses can use a diverse selection of diagnostic tests and assessment tools; however, different factors affect the validity and reliability of the results produced by these tests or tools. Nurses must be aware of these factors in order to select the most appropriate test or tool and to accurately interpret the results.

ORDER NOW FOR CUSTOMIZED SOLUTION PAPERS

Not only do these diagnostic tests affect adults, body measurements can provide a general picture of whether a child is receiving adequate nutrition or is at risk for health issues. These data, however, are just one aspect to be considered. Lifestyle, family history, and culture—among other factors—are also relevant. That said, gathering and communicating this information can be a delicate process. NURS 6512 Assignment 1: Assessment Tools and Diagnostic Tests in Adults and Children Case Study

Photo Credit: Getty Images/Hero Images

For this Assignment, you will consider the validity and reliability of different assessment tools and diagnostic tests. You will explore issues such as sensitivity, specificity, and positive and negative predictive values. You will also consider examples of children with various weight issues. You will explore how you could effectively gather information and encourage parents and caregivers to be proactive about their children’s health and weight.

To Prepare

  • Review this week’s Learning Resources and consider factors that impact the validity and reliability of various assessment tools and diagnostic tests. You also will review examples of pediatric patients and their families as it relates to BMI.
  • By Day 1 of this week, you will be assigned to one of the following Assignment options by your Instructor: Adult Assessment Tools or Diagnostic Tests (option 1), or Child Health Case (Option 2). Note: Please see the “Course Announcements” section of the classroom for your assignments from your Instructor.
  • Search the Walden Library and credible sources for resources explaining the tool or test you were assigned. What is its purpose, how is it conducted, and what information does it gather?
  • Also, as you search the Walden library and credible sources, consider what the literature discusses regarding the validity, reliability, sensitivity, specificity, predictive values, ethical dilemmas, and controversies related to the test or tool.
  • If you are assigned Assignment Option 2 (Child), consider what health issues and risks may be relevant to the child in the health example.
    • Based on the risks you identified, consider what further information you would need to gain a full understanding of the child’s health. Think about how you could gather this information in a sensitive fashion.
    • Consider how you could encourage parents or caregivers to be proactive toward the child’s health.

The Assignment

Assignment (3–4 pages, not including title and reference pages):

Assignment Option 1: Adult Assessment Tools or Diagnostic Tests:
Include the following:

  • A description of how the assessment tool or diagnostic test you were assigned is used in healthcare.
    • What is its purpose?
    • How is it conducted?
    • What information does it gather?
  • Based on your research, evaluate the test or the tool’s validity and reliability, and explain any issues with sensitivity, reliability, and predictive values. Include references in appropriate APA formatting.

Assignment Option 2: Child Health Case:
Include the following:

  • An explanation of the health issues and risks that are relevant to the child you were assigned.
  • Describe additional information you would need in order to further assess his or her weight-related health.
  • Identify and describe any risks and consider what further information you would need to gain a full understanding of the child’s health. Think about how you could gather this information in a sensitive fashion.
  • Taking into account the parents’ and caregivers’ potential sensitivities, list at least three specific questions you would ask about the child to gather more information.
  • Provide at least two strategies you could employ to encourage the parents or caregivers to be proactive about their child’s health and weight. NURS 6512 Assignment 1: Assessment Tools and Diagnostic Tests in Adults and Children Case Study
By Day 6 of Week 3

Submit your Assignment.

Submission and Grading Information

To submit your completed Assignment for review and grading, do the following:

  • Please save your Assignment using the naming convention “WK3Assgn1+last name+first initial.(extension)” as the name.
  • Click the Week 3 Assignment 1 Rubric to review the Grading Criteria for the Assignment.
  • Click the Week 3 Assignment 1 link. You will also be able to “View Rubric” for grading criteria from this area.
  • Next, from the Attach File area, click on the Browse My Computer button. Find the document you saved as “WK3Assgn1+last name+first initial.(extension)” and click Open.
  • If applicable: From the Plagiarism Tools area, click the checkbox for I agree to submit my paper(s) to the Global Reference Database.
  • Click on the Submit button to complete your submission.

 

Excellent Good Fair Poor
In 3–4 pages, address the following: 

A description of how the assessment tool or diagnostic test you were assigned is used in healthcare.
o What is its purpose?
o How is it conducted?
o What information does it gather?

30 (30%) – 35 (35%)
The response clearly, accurately, and with specific detail describes how the assessment tool or diagnostic test assigned is used in healthcare, including its purpose, how it is conducted, and what information it gathers.
24 (24%) – 29 (29%)
The response accurately describes how the assessment tool or diagnostic test assigned is used in healthcare, including its purpose, how it is conducted, and what information it gathers.
18 (18%) – 23 (23%)
The response vaguely and/or with some inaccuracy describes how the assessment tool or diagnostic test assigned is used in healthcare, including its purpose, how it is conducted, and what information it gathers.
(0%) – 17 (17%)
The response is inaccurate or missing descriptions of how the assessment tool or diagnostic test assigned is used in healthcare, including its purpose, how it is conducted, and what information it gathers.
Based on your research, evaluate the test or the tool’s validity and reliability, and explain any issues with sensitivity, reliability, and predictive values.
45 (45%) – 50 (50%)
The response accurately and thoroughly evaluates the test or tool’s validity and reliability, and explains any issues with clear sensitivity, reliability, and predictive values. Student’s research is clear, accurate, and appropriate for the evaluation.
39 (39%) – 44 (44%)
The response accurately evaluates the test or tool’s validity and reliability, and explains any issues with sensitivity, reliability, and predictive values. Student’s research is somewhat clear, accurate,and appropriate for the evaluation.
33 (33%) – 38 (38%)
The response vaguely and/or with some inaccuracy evaluates the test or tool’s validity and reliability, and explains any issues with some sensitivity, reliability, and predictive values. Student’s research is vague or inaccurate for the evaluation.
(0%) – 32 (32%)
“The response is inaccurate and/or missing evaluations of the test or tool’s validity and reliability, with explanations of any issues missing or lacking sensitivity, reliability, and predictive values. Student’s research is missing, inaccurate, or lacking for the evaluation.
Written Expression and Formatting – Paragraph Development and Organization:
Paragraphs make clear points that support well-developed ideas, flow logically, and demonstrate continuity of ideas. Sentences are carefully focused–neither long and rambling nor short and lacking substance. A clear and comprehensive purpose statement and introduction are provided that delineate all required criteria.
(5%) – 5 (5%)
Paragraphs and sentences follow writing standards for flow, continuity, and clarity. A clear and comprehensive purpose statement, introduction, and conclusion are provided that delineate all required criteria.
(4%) – 4 (4%)
Paragraphs and sentences follow writing standards for flow, continuity, and clarity 80% of the time. Purpose, introduction, and conclusion of the assignment are stated, yet are brief and not descriptive.
(3%) – 3 (3%)
Paragraphs and sentences follow writing standards for flow, continuity, and clarity 60%–79% of the time. Purpose, introduction, and conclusion of the assignment are vague or off topic.
(0%) – 2 (2%)
Paragraphs and sentences follow writing standards for flow, continuity, and clarity < 60% of the time. No purpose statement, introduction, or conclusion were provided.
Written Expression and Formatting – English writing standards:
Correct grammar, mechanics, and proper punctuation
(5%) – 5 (5%)
Uses correct grammar, spelling, and punctuation with no errors.
(4%) – 4 (4%)
Contains a few (1 or 2) grammar, spelling, and punctuation errors.
(3%) – 3 (3%)
Contains several (3 or 4) grammar, spelling, and punctuation errors.
(0%) – 2 (2%)
Contains many (≥ 5) grammar, spelling, and punctuation errors that interfere with the reader’s understanding.
Written Expression and Formatting – The paper follows correct APA format for title page, headings, font, spacing, margins, indentations, page numbers, running heads, parenthetical/in-text citations, and reference list.
(5%) – 5 (5%)
Uses correct APA format with no errors.
(4%) – 4 (4%)
Contains a few (1 or 2) APA format errors.
(3%) – 3 (3%)
Contains several (3 or 4) APA format errors.
(0%) – 2 (2%)
Contains many (≥ 5) APA format errors.
Total Points: 100
Excellent Good Fair Poor
In 3–4 pages, address the following: 

An explanation of the health issues and risks that are relevant to the child you were assigned.

20 (20%) – 25 (25%)
The response clearly, accurately, and in detail explains the relevant health issues and risks for the assigned child.
19 (19%) – 24 (24%)
The response accurately explains the relevant health issues and risks for the assigned child.
18 (18%) – 23 (23%)
The response vaguely and with some inaccuracy explains the relevant health issues and risks for the assigned child.
(0%) – 17 (17%)
The response is inaccurate and/or missing explanations of the relevant health issues and risks for the assigned child.
Describe additional information you would need in order to further assess his or her weight-related health.
20 (20%) – 25 (25%)
The response clearly and accurately describes detailed additional information needed to further assess the child’s weight-related health.
19 (19%) – 24 (24%)
The response accurately describes additional information needed to further assess the child’s weight-related health.
18 (18%) – 23 (23%)
The response vaguely and with some inaccuracy describes additional information needed to further assess the child’s weight-related health.
(0%) – 17 (17%)
The response is inaccurate and/or missing a description of additional information needed to further assess the child’s weight-related health.
Identify and describe any risks, and consider what further information you would need to gain a full understanding of the child’s health. Think about how you could gather this information in a sensitive fashion.
18 (18%) – 20 (20%)
The response clearly and accurately identifies and describes in detail any risks to the child’s health. The response clearly and accurately identifies and describes in detail further information needed to gain a full understanding of the child’s health, with a detailed explanation of how to gather that information in a way that is sensitive to the child.
16 (16%) – 17 (17%)
The response accurately identifies and describes any risks to the child’s health. The response accurately identifies and describes further information needed to gain a full understanding of the child’s health, with a clear explanation of how to gather that information in a way that is sensitive to the child. NURS 6512 Assignment 1: Assessment Tools and Diagnostic Tests in Adults and Children Case Study
14 (14%) – 14 (14%)
The response vaguely and with some inaccuracy identifies and describes any risks to the child’s health. The response vaguely identifies and describes further information needed to gain a full understanding of the child’s health, with a vague explanation of how to gather that information in a way that is sensitive to the child.
(0%) – 13 (13%)
The response identifies inaccurately and/or is missing descriptions of any risks to the child’s health. The response identifies inaccurately and/or is missing descriptions of further information needed to gain a full understanding of the child’s health, with an inadequate or missing explanation of how to gather that information in a way that is sensitive to the child.
Taking into account the parents’ and caregivers’ potential sensitivities, list at least three specific questions you would ask about the child to gather more information.
10 (10%) – 10 (10%)
The response clearly and accurately lists three or more specific questions that would gather more information about the child. Specific questions are carefully worded to clearly demonstrate sensitivity to the parent(s) or caregiver(s) of the child.
(9%) – 9 (9%)
The response lists three specific questions that would gather more information about the child. Specific questions are worded to demonstrate sensitivity to the parent(s) or caregiver(s) of the child.
(8%) – 8 (8%)
The response lists three questions with wording that is vague and lacking specificity for gathering more information about the child. Some wording of the questions lacks sensitivity to the parent(s) or caregiver(s) of the child.
(0%) – 7 (7%)
The response lists two or fewer confusing or inadequate questions, or is missing questions, for gathering more information about the child. Wording of questions provided lacks sensitivity to the parent(s) or caregiver(s) of the child.
Provide at least two strategies you could employ to encourage the parents or caregivers to be proactive about their child’s health and weight.
(5%) – 5 (5%)
The response clearly describes two or more detailed strategies to encourage the parent(s) or caregiver(s) to be proactive about the child’s health and weight.
(4%) – 4 (4%)
The response describes at least two strategies to encourage the parent(s) or caregiver(s) to be proactive about the child’s health and weight.
(3%) – 3 (3%)
The response vaguely describes two strategies to encourage the parent(s) or caregiver(s) to be proactive about the child’s health and weight.
(0%) – 2 (2%)
The response inadequately describes one strategy or is missing strategies to encourage the parent(s) or caregiver(s) to be proactive about the child’s health and weight.
Written Expression and Formatting – Paragraph Development and Organization:
Paragraphs make clear points that support well-developed ideas, flow logically, and demonstrate continuity of ideas. Sentences are carefully focused–neither long and rambling nor short and lacking substance. A clear and comprehensive purpose statement and introduction are provided that delineate all required criteria.
(5%) – 5 (5%)
Paragraphs and sentences follow writing standards for flow, continuity, and clarity. A clear and comprehensive purpose statement, introduction, and conclusion are provided that delineate all required criteria.
(4%) – 4 (4%)
Paragraphs and sentences follow writing standards for flow, continuity, and clarity 80% of the time. Purpose, introduction, and conclusion of the assignment are stated, yet are brief and not descriptive.
(3%) – 3 (3%)
Paragraphs and sentences follow writing standards for flow, continuity, and clarity 60%–79% of the time. Purpose, introduction, and conclusion of the assignment are vague or off topic.
(0%) – 2 (2%)
Paragraphs and sentences follow writing standards for flow, continuity, and clarity < 60% of the time. No purpose statement, introduction, or conclusion were provided.
Written Expression and Formatting – English writing standards:
Correct grammar, mechanics, and proper punctuation
(5%) – 5 (5%)
Uses correct grammar, spelling, and punctuation with no errors.
(4%) – 4 (4%)
Contains a few (1 or 2) grammar, spelling, and punctuation errors.
(3%) – 3 (3%)
Contains several (3 or 4) grammar, spelling, and punctuation errors.
(0%) – 2 (2%)
Contains many (≥ 5) grammar, spelling, and punctuation errors that interfere with the reader’s understanding.
Written Expression and Formatting – The paper follows correct APA format for title page, headings, font, spacing, margins, indentations, page numbers, running heads, parenthetical/in-text citations, and reference list.
(5%) – 5 (5%)
Uses correct APA format with no errors.
(4%) – 4 (4%)
Contains a few (1 or 2) APA format errors.
(3%) – 3 (3%)
Contains several (3 or 4) APA format errors.
(0%) – 2 (2%)
Contains many (≥ 5) APA format errors.
Total Points: 100

Assignment 2: Digital Clinical Experience (DCE): Health History Assessment

Assignment 2: Digital Clinical Experience (DCE): Health History Assessment

A comprehensive health history is essential to providing quality care for patients across the lifespan, as it helps to properly identify health risks, diagnose patients, and develop individualized treatment plans. To effectively collect these heath histories, you must not only have strong communication skills, but also the ability to quickly establish trust and confidence with your patients. For this DCE Assignment, you begin building your communication and assessment skills as you collect a health history from a volunteer “patient.”

ORDER NOW FOR CUSTOMIZED SOLUTION PAPERS

 

Photo Credit: Getty Images/Caiaimage

To Prepare

  • Review this week’s Learning Resources as well as the Taking a Health History media program, and consider how you might incorporate these strategies. Download and review the Student Checklist: Health History Guide and the History Subjective Data Checklist, provided in this week’s Learning Resources, to guide you through the necessary components of the assessment. Assignment 2: Digital Clinical Experience (DCE): Health History Assessment
  • Access and login to Shadow Health using the link in the left-hand navigation of the Blackboard classroom.
  • Review the Shadow Health Student Orientation media program and the Useful Tips and Tricks document provided in the week’s Learning Resources to guide you through Shadow Health.
  • Review the Week 4 DCE Health History Assessment Rubric, provided in the Assignment submission area, for details on completing the Assignment.

DCE Health History Assessment:

Complete the following in Shadow Health:

Orientation

  • DCE Orientation (15 minutes)
  • Conversation Concept Lab (50 minutes)

Health History

  • Health History of Tina Jones (180 minutes)

Note: Each Shadow Health Assessment may be attempted and reopened as many times as necessary prior to the due date to achieve 80% or better, but you must take all attempts by the Week 4 Day 7 deadline.

Submission and Grading Information

No Assignment submission due this week but will be due Day 7, Week 4.

 

Excellent Good Fair Poor
Student DCE score 

(DCE percentages will be calculated automatically by Shadow Health after the assignment is completed.)

Note: DCE Score – Do not round up on the DCE score.

56 (56%) – 60 (60%)
DCE score>93
51 (51%) – 55 (55%)
DCE Score 86-92
46 (46%) – 50 (50%)
DCE Score 80-85
(0%) – 45 (45%)
DCE Score <79 

No DCE completed.

Subjective Documentation in Provider Notes 

Subjective narrative documentation in Provider Notes is detailed and organized and includes:

Chief Complaint (CC), HPI, Current Medications, Allergies, Past Medical History, Family History, Social History and Review of Systems (ROS)

ROS: covers all body systems that may help you formulate a list of differential diagnoses. You should list each system as follows:
General: Head: EENT: etc.

You should list these in bullet format and document the systems in order from head to toe.

36 (36%) – 40 (40%)
Documentation is detailed and organized with all pertinent information noted in professional language. 

Documentation includes all pertinent documentation to include Chief Complaint (CC), HPI, Current Medications, Allergies, Past Medical History, Family History, Social History and Review of Systems (ROS).

31 (31%) – 35 (35%)
Documentation with sufficient details, some organization and some pertinent information noted in professional language. Assignment 2: Digital Clinical Experience (DCE): Health History Assessment 

Documentation provides some of the Chief Complaint (CC), HPI, Current Medications, Allergies, Past Medical History, Family History, Social History and Review of Systems (ROS).

26 (26%) – 30 (30%)
Documentation with inadequate details and/or organization; and inadequate pertinent information noted in professional language. 

Limited or/minimum documentation provided to analyze students critical thinking abilities for the Chief Complaint (CC), HPI, Current Medications, Allergies, Past Medical History, Family History, Social History and Review of Systems (ROS).

(0%) – 25 (25%)
Documentation lacks any details and/or organization; and does not provide pertinent information noted in professional language. 

No information is provided for the Chief Complaint (CC), HPI, Current Medications, Allergies, Past Medical History, Family History, Social History and Review of Systems (ROS).

or

No documentation provided.

Total Points: 100

Assignment 1: Lab Assignment: Differential Diagnosis for Skin Conditions

Assignment 1: Lab Assignment: Differential Diagnosis for Skin Conditions

Properly identifying the cause and type of a patient’s skin condition involves a process of elimination known as differential diagnosis. Using this process, a health professional can take a given set of physical abnormalities, vital signs, health assessment findings, and patient descriptions of symptoms, and incrementally narrow them down until one diagnosis is determined as the most likely cause.

In this Lab Assignment, you will examine several visual representations of various skin conditions, describe your observations, and use the techniques of differential diagnosis to determine the most likely condition. Assignment 1: Lab Assignment: Differential Diagnosis for Skin Conditions

To Prepare

  • Review the Skin Conditions document provided in this week’s Learning Resources, and select one condition to closely examine for this Lab Assignment.
  • Consider the abnormal physical characteristics you observe in the graphic you selected. How would you describe the characteristics using clinical terminologies?
  • Explore different conditions that could be the cause of the skin abnormalities in the graphics you selected.
  • Consider which of the conditions is most likely to be the correct diagnosis, and why.
  • Search the Walden library for one evidence-based practice, peer-reviewed article based on the skin condition you chose for this Lab Assignment. Assignment 1: Lab Assignment: Differential Diagnosis for Skin Conditions
  • Review the Comprehensive SOAP Exemplar found in this week’s Learning Resources to guide you as you prepare your SOAP note.
  • Download the SOAP Template found in this week’s Learning Resources, and use this template to complete this Lab Assignment.

ORDER NOW FOR CUSTOMIZED SOLUTION PAPERS

The Lab Assignment

  • Choose one skin condition graphic (identify by number in your Chief Complaint) to document your assignment in the SOAP (Subjective, Objective, Assessment, and Plan) note format rather than the traditional narrative style. Refer to Chapter 2 of the Sullivan text and the Comprehensive SOAP Template in this week’s Learning Resources for guidance. Remember that not all comprehensive SOAP data are included in every patient case.
  • Use clinical terminologies to explain the physical characteristics featured in the graphic. Formulate a differential diagnosis of three to five possible conditions for the skin graphic that you chose. Determine which is most likely to be the correct diagnosis and explain your reasoning using at least three different references, one reference from current evidence-based literature from your search and two different references from this week’s Learning Resources. Assignment 1: Lab Assignment: Differential Diagnosis for Skin Conditions
By Day 7 of Week 4

Submit your Lab Assignment.

Submission and Grading Information

To submit your completed Assignment for review and grading, do the following:

  • Please save your Assignment using the naming convention “WK4Assgn1+last name+first initial.(extension)” as the name.
  • Click the Week 4 Assignment 1 Rubric to review the Grading Criteria for the Assignment.
  • Click the Week 4 Assignment 1 link. You will also be able to “View Rubric” for grading criteria from this area.
  • Next, from the Attach File area, click on the Browse My Computer button. Find the document you saved as “WK4Assgn1+last name+first initial.(extension)” and click Open.
  • If applicable: From the Plagiarism Tools area, click the checkbox for I agree to submit my paper(s) to the Global Reference Database.
  • Click on the Submit button to complete your submission.
Excellent Good Fair Poor
Using the SOAP (Subjective, Objective, Assessment, and Plan) note format:
·  Create documentation, following SOAP format, of your assignment to choose one skin condition graphic (identify by number in your Chief Complaint). ·   Use clinical terminologies to explain the physical characteristics featured in the graphic.
30 (30%) – 35 (35%)
The response clearly, accurately, and thoroughly follows the SOAP format to document one skin condition graphic and accurately identifies the graphic by number in the Chief Complaint. The response clearly and thoroughly explains all physical characteristics featured in the graphic using accurate terminologies.
24 (24%) – 29 (29%)
The response accurately follows the SOAP format to document one skin condition graphic and accurately identifies the graphic by number in the Chief Complaint. The response explains most physical characteristics featured in the graphic using accurate terminologies.
18 (18%) – 23 (23%)
The response follows the SOAP format, with vagueness and some inaccuracy in documenting one skin condition graphic, and accurately identifies the graphic by number in the Chief Complaint. The response explains some physical characteristics featured in the graphic using mostly accurate terminologies.
(0%) – 17 (17%)
The response inaccurately follows the SOAP format or is missing documentation for one skin condition graphic and is missing or inaccurately identifies the graphic by number in the Chief Complaint. The response explains some or few physical characteristics featured in the graphic using terminologies with multiple inaccuracies.
·   Formulate a different diagnosis of three to five possible considerations for the skin graphic.    ·   Determine which is most likely to be the correct diagnosis, and explain your reasoning using at least three different references from current evidence-based literature.
45 (45%) – 50 (50%)
The response clearly, thoroughly, and accurately formulates a different diagnosis of five possible considerations for the skin graphic. The response determines the most likely correct diagnosis with reasoning that is explained clearly, accurately, and thoroughly using three or more different references from current evidence-based literature.
39 (39%) – 44 (44%)
The response accurately formulates a different diagnosis of three to five possible considerations for the skin graphic. The response determines the most likely correct diagnosis with reasoning that is explained accurately using at least three different references from current evidence-based literature.
33 (33%) – 38 (38%)
The response vaguely or with some inaccuracy formulates a different diagnosis of three possible considerations for the skin graphic. The response determines the most likely correct diagnosis with reasoning that is explained vaguely and with some inaccuracy using three different references from current evidence-based literature.
(0%) – 32 (32%)
The response formulates inaccurately, incompletely, or is missing a different diagnosis of possible considerations for the skin graphic, with two or fewer possible considerations provided. The response vaguely, inaccurately, or incompletely determines the most likely correct diagnosis with reasoning that is missing or explained using two or fewer different references from current evidence-based literature.
Written Expression and Formatting – Paragraph Development and Organization:
Paragraphs make clear points that support well-developed ideas, flow logically, and demonstrate continuity of ideas. Sentences are carefully focused–neither long and rambling nor short and lacking substance. A clear and comprehensive purpose statement and introduction are provided that delineate all required criteria.
(5%) – 5 (5%)
Paragraphs and sentences follow writing standards for flow, continuity, and clarity. A clear and comprehensive purpose statement, introduction, and conclusion are provided that delineate all required criteria.
(4%) – 4 (4%)
Paragraphs and sentences follow writing standards for flow, continuity, and clarity 80% of the time. Purpose, introduction, and conclusion of the assignment are stated, yet are brief and not descriptive.
(3%) – 3 (3%)
Paragraphs and sentences follow writing standards for flow, continuity, and clarity 60%–79% of the time. Purpose, introduction, and conclusion of the assignment are vague or off topic.
(0%) – 2 (2%)
Paragraphs and sentences follow writing standards for flow, continuity, and clarity < 60% of the time. No purpose statement, introduction, or conclusion were provided.
Written Expression and Formatting – English writing standards:
Correct grammar, mechanics, and proper punctuation
(5%) – 5 (5%)
Uses correct grammar, spelling, and punctuation with no errors.
(4%) – 4 (4%)
Contains a few (1 or 2) grammar, spelling, and punctuation errors.
(3%) – 3 (3%)
Contains several (3 or 4) grammar, spelling, and punctuation errors.
(0%) – 2 (2%)
Contains many (≥ 5) grammar, spelling, and punctuation errors that interfere with the reader’s understanding.
Written Expression and Formatting – The paper follows correct APA format for title page, headings, font, spacing, margins, indentations, page numbers, running heads, parenthetical/in-text citations, and reference list.
(5%) – 5 (5%)
Uses correct APA format with no errors.
(4%) – 4 (4%)
Contains a few (1 or 2) APA format errors. Assignment 1: Lab Assignment: Differential Diagnosis for Skin Conditions
(3%) – 3 (3%)
Contains several (3 or 4) APA format errors.
(0%) – 2 (2%)
Contains many (≥ 5) APA format errors.
Total Points: 100

NURS 6051N WEEK 1 Discussion: The Application of Data to Problem-Solving

NURS 6051N WEEK 1 Discussion: The Application of Data to Problem-Solving

Discussion: The Application of Data to Problem-Solving
In the modern era, there are few professions that do not to some extent rely on data. Stockbrokers rely on market data to advise clients on financial matters. Meteorologists rely on weather data to forecast weather conditions, while realtors rely on data to advise on the purchase and sale of property. In these and other cases, data not only helps solve problems, but adds to the practitioner’s and the discipline’s body of knowledge.

Of course, the nursing profession also relies heavily on data. The field of nursing informatics aims to make sure nurses have access to the appropriate date to solve healthcare problems, make decisions in the interest of patients, and add to knowledge. NURS 6051N WEEK 1 Discussion: The Application of Data to Problem-Solving

In this Discussion, you will consider a scenario that would benefit from access to data and how such access could facilitate both problem-solving and knowledge formation.

ORDER NOW FOR CUSTOMIZED SOLUTION PAPERS

To Prepare:

Reflect on the concepts of informatics and knowledge work as presented in the Resources.
Consider a hypothetical scenario based on your own healthcare practice or organization that would require or benefit from the access/collection and application of data. Your scenario may involve a patient, staff, or management problem or gap.
By Day 3 of Week 1
Post a description of the focus of your scenario. Describe the data that could be used and how the data might be collected and accessed. What knowledge might be derived from that data? How would a nurse leader use clinical reasoning and judgment in the formation of knowledge from this experience?

By Day 6 of Week 1
Respond to at least two of your colleagues* on two different days, asking questions to help clarify the scenario and application of data, or offering additional/alternative ideas for the application of nursing informatics principles.

*Note: Throughout this program, your fellow students are referred to as colleagues.

Submission and Grading Information

NURS_5051_Module01_Week01_Discussion_Rubric

Grid View
List View
Excellent Good Fair Poor
Main Posting
45 (45%) – 50 (50%)
Answers all parts of the discussion question(s) expectations with reflective critical analysis and synthesis of knowledge gained from the course readings for the module and current credible sources.
Supported by at least three current, credible sources.

Written clearly and concisely with no grammatical or spelling errors and fully adheres to current APA manual writing rules and style.

40 (40%) – 44 (44%)
Responds to the discussion question(s) and is reflective with critical analysis and synthesis of knowledge gained from the course readings for the module.
At least 75% of post has exceptional depth and breadth. NURS 6051N WEEK 1 Discussion: The Application of Data to Problem-Solving

Supported by at least three credible sources.

Written clearly and concisely with one or no grammatical or spelling errors and fully adheres to current APA manual writing rules and style.

35 (35%) – 39 (39%)
Responds to some of the discussion question(s).
One or two criteria are not addressed or are superficially addressed.

Is somewhat lacking reflection and critical analysis and synthesis.

Somewhat represents knowledge gained from the course readings for the module.

Post is cited with two credible sources.

Written somewhat concisely; may contain more than two spelling or grammatical errors.

Contains some APA formatting errors.

0 (0%) – 34 (34%)
Does not respond to the discussion question(s) adequately.
Lacks depth or superficially addresses criteria.

Lacks reflection and critical analysis and synthesis.

Does not represent knowledge gained from the course readings for the module.

Contains only one or no credible sources.

Not written clearly or concisely.

Contains more than two spelling or grammatical errors.

Does not adhere to current APA manual writing rules and style.

Main Post: Timeliness
10 (10%) – 10 (10%)
Posts main post by day 3.
0 (0%) – 0 (0%)
0 (0%) – 0 (0%)
0 (0%) – 0 (0%)
Does not post by day 3.
First Response
17 (17%) – 18 (18%)
Response exhibits synthesis, critical thinking, and application to practice settings.
Responds fully to questions posed by faculty.

Provides clear, concise opinions and ideas that are supported by at least two scholarly sources.

Demonstrates synthesis and understanding of learning objectives.

Communication is professional and respectful to colleagues.

Responses to faculty questions are fully answered, if posed.

Response is effectively written in standard, edited English.

15 (15%) – 16 (16%)
Response exhibits critical thinking and application to practice settings.
Communication is professional and respectful to colleagues.

Responses to faculty questions are answered, if posed.

Provides clear, concise opinions and ideas that are supported by two or more credible sources.

Response is effectively written in standard, edited English.

13 (13%) – 14 (14%)
Response is on topic and may have some depth.
Responses posted in the discussion may lack effective professional communication.

Responses to faculty questions are somewhat answered, if posed.

Response may lack clear, concise opinions and ideas, and a few or no credible sources are cited.

0 (0%) – 12 (12%)
Response may not be on topic and lacks depth.
Responses posted in the discussion lack effective professional communication.

Responses to faculty questions are missing.

No credible sources are cited.

Second Response
16 (16%) – 17 (17%)
Response exhibits synthesis, critical thinking, and application to practice settings.
Responds fully to questions posed by faculty.

Provides clear, concise opinions and ideas that are supported by at least two scholarly sources.

Demonstrates synthesis and understanding of learning objectives.

Communication is professional and respectful to colleagues.

Responses to faculty questions are fully answered, if posed.

Response is effectively written in standard, edited English.

14 (14%) – 15 (15%)
Response exhibits critical thinking and application to practice settings.
Communication is professional and respectful to colleagues.

Responses to faculty questions are answered, if posed.

Provides clear, concise opinions and ideas that are supported by two or more credible sources.

Response is effectively written in standard, edited English.

12 (12%) – 13 (13%)
Response is on topic and may have some depth.
Responses posted in the discussion may lack effective professional communication.

Responses to faculty questions are somewhat answered, if posed. NURS 6051N WEEK 1 Discussion: The Application of Data to Problem-Solving

Response may lack clear, concise opinions and ideas, and a few or no credible sources are cited.

0 (0%) – 11 (11%)
Response may not be on topic and lacks depth.
Responses posted in the discussion lack effective professional communication.

Responses to faculty questions are missing.

No credible sources are cited.

Participation
5 (5%) – 5 (5%)
Meets requirements for participation by posting on three different days.
0 (0%) – 0 (0%)
0 (0%) – 0 (0%)
0 (0%) – 0 (0%)
Does not meet requirements for participation by posting on 3 different days.

Application of Data to Problem-Solving discussion

Application of Data to Problem-Solving discussion

NURS 6051 – The Impact of Nursing Informatics on Patient Outcomes and Patient Care Efficiencies

NURS 6051 – The Impact of Nursing Informatics on Patient Outcomes and Patient Care Efficiencies

The Assignment: (4-5 pages)

In a 4- to 5-page project proposal written to the leadership of your healthcare organization, propose a nursing informatics project for your organization that you advocate to improve patient outcomes or patient-care efficiency. Your project proposal should include the following:

  • Describe the project you propose.
  • Identify the stakeholders impacted by this project.
  • Explain the patient outcome(s) or patient-care efficiencies this project is aimed at improving and explain how this improvement would occur. Be specific and provide examples.
  • Identify the technologies required to implement this project and explain why.
  • Identify the project team (by roles) and explain how you would incorporate the nurse informaticist in the project team. NURS 6051 – The Impact of Nursing Informatics on Patient Outcomes and Patient Care Efficiencies

ORDER NOW FOR CUSTOMIZED SOLUTION PAPERS

Excellent Good Fair Poor
In a 4- to 5-page project proposal written to the leadership of your healthcare organization, propose a nursing informatics project for your organization that you advocate to improve patient outcomes or patient care efficiency. Your project proposal should include the following: 

·   Describe the project you propose.

·   Identify the stakeholders impacted by this project.

·   Explain the patient outcome(s) or patient-care efficiencies this project is aimed at improving, and explain how this improvement would occur. Be specific and provide examples.

·   Identify the technologies required to implement this project and explain why.

·   Identify the project team (by roles) and explain how you would incorporate the nurse informaticist in the project team.

77 (77%) – 85 (85%)
The response accurately and thoroughly describes in detail the project proposed. 

The response accurately and clearly identifies the stakeholders impacted by the project proposed.

The response accurately and thoroughly explains in detail the patient outcome(s) or patient-care efficiencies that the project proposed is aimed at improving, including an accurate and detailed explanation, with sufficient supporting evidence of how this improvement would occur.

The response accurately and clearly identifies the technologies required to implement the project proposed with a detailed explanation why.

The response accurately and clearly identifies the project team (by roles) and thoroughly explains in detail how to incorporate the nurse informaticist in the project team.

Includes: 3 or more peer-reviewed sources and 2 or more course resources.

68 (68%) – 76 (76%)
The response describes the project proposed. 

The response identifies the stakeholders impacted by the project proposed.

The response explains the patient outcome(s) or patient-care efficiencies that the project proposed is aimed at improving, including an explanation, with some supporting evidence of how this improvement would occur.

The response identifies the technologies required to implement the project proposed with an explanation why.

The response identifies the project team (by roles) and explains how to incorporate the nurse informaticist in the project team.

Includes: 2 peer-reviewed sources and 2 course resources.

60 (60%) – 67 (67%)
The response describing the project proposed is vague or inaccurate. 

The response identifying the stakeholders impacted by the project proposed is vague or inaccurate.

The response explaining the patient outcome(s) or patient-care efficiencies the project proposed is aimed at improving, including an explanation of how this improvement would occur, is vague or inaccurate, or includes little to no supporting evidence.

The response identifying the technologies required to implement the project proposed with an explanation why is vague or inaccurate.

The response identifying the project team (by roles) and an explanation of how to incorporate the nurse informaticist in the project team is vague or inaccurate.

Includes: 1 peer-reviewed sources and 1 course resources.

(0%) – 59 (59%)
The response describing the project proposed is vague and inaccurate, or is missing. 

The response identifying the stakeholders impacted by the project proposed is vague and inaccurate, or is missing.

The response explaining the patient outcome(s) or patient-care efficiencies the project proposed is aimed at improving, including an explanation of how this improvement would occur, is vague and inaccurate, includes no supporting evidence, or is missing.

The response identifying the technologies required to implement the project proposed with an explanation why is vague and inaccurate, or is missing.

The response identifying the project team (by roles) and an explanation of how to incorporate the nurse informaticist in the project team is vague and inaccurate, or is missing.

Includes: 1 or fewer resources.

Written Expression and Formatting – Paragraph Development and Organization: 

Paragraphs make clear points that support well developed ideas, flow logically, and demonstrate continuity of ideas. Sentences are carefully focused–neither long and rambling nor short and lacking substance.

(5%) – 5 (5%)
Paragraphs and sentences follow writing standards for flow, continuity, and clarity.
(4%) – 4 (4%)
Paragraphs and sentences follow writing standards for flow, continuity, and clarity 80% of the time.
3.5 (3.5%) – 3.5 (3.5%)
Paragraphs and sentences follow writing standards for flow, continuity, and clarity 60%- 79% of the time.
(0%) – 3 (3%)
Paragraphs and sentences follow writing standards for flow, continuity, and clarity < 60% of the time.
Written Expression and Formatting – English writing standards: 

Correct grammar, mechanics, and proper punctuation

(5%) – 5 (5%)
Uses correct grammar, spelling, and punctuation with no errors.
(4%) – 4 (4%)
Contains a few (1-2) grammar, spelling, and punctuation errors.
3.5 (3.5%) – 3.5 (3.5%)
Contains several (3-4) grammar, spelling, and punctuation errors.
(0%) – 3 (3%)
Contains many (≥ 5) grammar, spelling, and punctuation errors that interfere with the reader’s understanding.
Written Expression and Formatting – The paper follows correct APA format for title page, headings, font, spacing, margins, indentations, page numbers, running head, parenthetical/in-text citations, and reference list.
(5%) – 5 (5%)
Uses correct APA format with no errors.
(4%) – 4 (4%)
Contains a few (1-2) APA format errors.
3.5 (3.5%) – 3.5 (3.5%)
Contains several (3-4) APA format errors.
(0%) – 3 (3%)
Contains many (≥ 5) APA format errors.
Total Points: 100

 

Module 2: The Role of the Informatics Specialist in Healthcare (Weeks 3-4)

Laureate Education (Producer). (2018). The Nurse Informaticist [Video file]. Baltimore, MD: Author.

 

Learning Objectives

Students will:
  • Analyze interactions between nurse informaticists, data/technology specialists, and other professionals in healthcare organizations
  • Recommend strategies to improve interactions between nurse informaticists and other professionals
  • Analyze how nursing informatics as a specialty and new technologies impact interactions between nurse informaticists and members of healthcare teams
  • Recommend nursing informatics projects to improve outcomes or efficiencies in healthcare organizations
  • Identify stakeholders impacted by nursing informatics projects
  • Analyze how nursing informatics projects improve outcomes or efficiencies in healthcare organizations
  • Identify technologies required for implementation of nursing informatics projects
  • Analyze the role of the nurse informaticist in nursing informatics project teams
Due By Assignment
Week 3, Days 1–2 Read/Watch/Listen to the Learning Resources.
Compose your initial Discussion post.
Week 3, Day 3 Post your initial Discussion post.
Begin to compose your Assignment.
Week 3, Days 4-5 Review peer Discussion posts.
Compose your peer Discussion responses.
Continue to compose your Assignment.
Week 3, Day 6 Post at least two peer Discussion responses on two different days (and not the same day as the initial post).
Week 3, Day 7 Wrap up Discussion.
Deadline to submit your Assignment.
Week 4, Days 1-6 Continue to compose your Assignment
Week 4, Day 7 Deadline to submit your Assignment

Learning Resources

Note: To access this module’s required library resources, please click on the link to the Course Readings List, found in the Course Materials section of your Syllabus. NURS 6051 – The Impact of Nursing Informatics on Patient Outcomes and Patient Care Efficiencies

Required Readings

McGonigle, D., & Mastrian, K. G. (2017). Nursing informatics and the foundation of knowledge (4th ed.). Burlington, MA: Jones & Bartlett Learning.

  • Chapter 25, “The Art of Caring in Technology-Laden Environments” (pp. 525–535)
  • Chapter 26, “Nursing Informatics and the Foundation of Knowledge” (pp. 537–551)

Discussion: Interaction Between Nurse Informaticists and Other Specialists

Nature offers many examples of specialization and collaboration. Ant colonies and bee hives are but two examples of nature’s sophisticated organizations. Each thrives because their members specialize by tasks, divide labor, and collaborate to ensure food, safety, and general well-being of the colony or hive.

Of course, humans don’t fare too badly in this regard either. And healthcare is a great example. As specialists in the collection, access, and application of data, nurse informaticists collaborate with specialists on a regular basis to ensure that appropriate data is available to make decisions and take actions to ensure the general well-being of patients.

In this Discussion, you will reflect on your own observations of and/or experiences with informaticist collaboration. You will also propose strategies for how these collaborative experiences might be improved.

To Prepare:

  • Review the Resources and reflect on the evolution of nursing informatics from a science to a nursing specialty.
  • Consider your experiences with nurse Informaticists or technology specialists within your healthcare organization.

By Day 3 of Week 3

Post a description of experiences or observations about how nurse informaticists and/or data or technology specialists interact with other professionals within your healthcare organization. Suggest at least one strategy on how these interactions might be improved. Be specific and provide examples. Then, explain the impact you believe the continued evolution of nursing informatics as a specialty and/or the continued emergence of new technologies might have on professional interactions.

By Day 6 of Week 3

Respond to at least two of your colleagues* on two different days, offering one or more additional interaction strategies in support of the examples/observations shared or by offering further insight to the thoughts shared about the future of these interactions.

*Note: Throughout this program, your fellow students are referred to as colleagues.

Submission and Grading Information

Grading Criteria

To access your rubric:

Week 3 Discussion Rubric

Post by Day 3 and Respond by Day 6 of Week 3

To participate in this Discussion:

Week 3 Discussion


Assignment: The Impact of Nursing Informatics on Patient Outcomes and Patient Care Efficiencies

In the Discussion for this module, you considered the interaction of nurse informaticists with other specialists to ensure successful care. How is that success determined?

Patient outcomes and the fulfillment of care goals is one of the major ways that healthcare success is measured. Measuring patient outcomes results in the generation of data that can be used to improve results. Nursing informatics can have a significant part in this process and can help to improve outcomes by improving processes, identifying at-risk patients, and enhancing efficiency. NURS 6051 – The Impact of Nursing Informatics on Patient Outcomes and Patient Care Efficiencies

To Prepare:

  • Review the concepts of technology application as presented in the Resources.
  • Reflect on how emerging technologies such as artificial intelligence may help fortify nursing informatics as a specialty by leading to increased impact on patient outcomes or patient care efficiencies.

The Assignment: (4-5 pages)

In a 4- to 5-page project proposal written to the leadership of your healthcare organization, propose a nursing informatics project for your organization that you advocate to improve patient outcomes or patient-care efficiency. Your project proposal should include the following:

  • Describe the project you propose.
  • Identify the stakeholders impacted by this project.
  • Explain the patient outcome(s) or patient-care efficiencies this project is aimed at improving and explain how this improvement would occur. Be specific and provide examples.
  • Identify the technologies required to implement this project and explain why.
  • Identify the project team (by roles) and explain how you would incorporate the nurse informaticist in the project team.

By Day 7 of Week 4

Submit your completed Project Proposal.

Submission and Grading Information

To submit your completed Assignment for review and grading, do the following:

  • Please save your Assignment using the naming convention “WK4Assgn+last name+first initial.(extension)” as the name.
  • Click the Week 4 Assignment Rubric to review the Grading Criteria for the Assignment.
  • Click the Week 4 Assignment link. You will also be able to “View Rubric” for grading criteria from this area.
  • Next, from the Attach File area, click on the Browse My Computer button. Find the document you saved as “WK4Assgn+last name+first initial.(extension)” and click Open.
  • If applicable: From the Plagiarism Tools area, click the checkbox for I agree to submit my paper(s) to the Global Reference Database.
  • Click on the Submit button to complete your submission.
Grading Criteria

To access your rubric:

Week 4 Assignment Rubric

Check Your Assignment Draft for Authenticity

To check your Assignment draft for authenticity:

Submit your Week 4 Assignment draft and review the originality report.

Submit Your Assignment by Day 7 of Week 4

To participate in this Assignment:

Week 4 Assignment

NURS 6501 Advanced Pathophysiology Module 4 Knowledge Check

NURS 6501 Advanced Pathophysiology Module 4 Knowledge Check

  • Question 1

Needs Grading

A 67-year-old Caucasian woman was brought to the clinic by her son who stated that his mother had become slightly confused over the past several days. She had been stumbling at home and had fallen once but was able to ambulate with some difficulty. She had no other obvious problems and had been eating and drinking. The son became concerned when she forgot her son’s name, so he thought he better bring her to the clinic. 

PMH-Type II diabetes mellitus (DM) with peripheral neuropathy x 20 years. COPD. Depression after death of spouse several months ago

Social/family hx – non contributary except for 30 pack/year history tobacco use.

Meds: Metformin 500 mg po BID, ASA 81 mg po qam, escitalopram (Lexapro) 5 mg po q am started 2 months ago

Labs-CBC WNL; Chem 7- Glucose-92 mg/dl, BUN 18 mg/dl, Creatinine 1.1 mg/dl, Na+120 mmol/L,

K+4.2 mmol/L, CO237 m mol/L, Cl97 mmol/L.

The APRN refers the patient to the ED and called endocrinology for a consult for diagnosis and management of syndrome of inappropriate antidiuretic hormone (SIADH).

 

Question:

Define SIADH and identify any patient characteristics that may have contributed to the development of SIADH.

 

Correct Answer:  

SIADH is a group of symptoms that occurs when antidiuretic hormone (ADH, arginine vasopressin) is secreted in the absence of osmotic or physiologic stimuli. These stimuli include: Increased serum osmolality, decreased plasma volume, and hypotension. A decrease in plasma osmolality normally inhibits ADH production and secretion. SIADH is characterized by fluid retention, dilutional hyponatremia, hypochloremia, concentrated urine, and lack of intravascular volume depletion. SIADH is characterized by normal to increased blood volume in normoproteinemia, nonedematous, and hyponatremic patients with normal renal and endocrine function. ADH regulates the body’s water balance. It is synthesized in the hypothalamus and stored in the posterior pituitary gland. When released into the circulation, it acts on the kidney’s distal tubules and collecting ducts, increasing their permeability to water. This decreases urine volume because more water is being reabsorbed and returned to the circulation. It also serves to produce more concentrated urine.

  • Question 2

Needs Grading

A 43-year-old female presents to the clinic with a chief complaint of fever, chills, nausea and vomiting and weakness. She has been unable to keep any food, liquids or medications down. The symptoms began 3 days ago and have not responded to ibuprofen, acetaminophen, or Nyquil when she tried to take them. The temperature has reached as high as 102˚F. 

 

Allergies: none known to drugs or food or environmental

 

Medications-20 mg prednisone po qd, omeprazole 10 po qam

 

PMH-significant for 20-year history of steroid dependent rheumatoid arthritis (RA). GERD. No other significant illnesses or surgeries.

 

Social-denies alcohol, illicit drugs, vaping, tobacco use

 

Physical exam

 

Thin, ill appearing woman who is sitting in exam room chair as she said she was too weak to climb on the exam table. VS Temp 101.2˚F, BP 98/64, pulse 110, Resp 16, PaO2 96% on room air.

 

ROS negative other than GI symptoms.

 

Based on the patient’s clinical presentation, the APRN diagnoses the patient as having secondary hypocortisolism due to the lack of prednisone the patient was taking for her RA secondary to vomiting.

 

Question:

 

Explain why the patient exhibited these symptoms? 

Correct Answer:  

An adrenal insufficiency requires some type of trigger or stressor such as surgery, trauma, infection or acute withdrawal of glucocorticoids. The patient had several factors contributing to her present situation. She had RA x 20 years that necessitated oral prednisone. Increased levels of both glucocorticoids (primarily cortisol) and mineralocorticoids (primarily aldosterone) are needed for the body to adapt to the stress Corticotropin-releasing hormone (CRH) from the hypothalamus eventually prompts release of ACTH from the anterior pituitary gland. ACTH then stimulates release and 3 synthesis of cortisol from the adrenal cortex. Cortisol mobilizes amino acids from skeletal muscle and generally enhances the liver’s capacity for gluconeogenesis as well as enhances normal immune activity and maintenance of cardiovascular integrity. It also influences fat, carbohydrate and protein. Catecholamines cause vasoconstriction, which in the kidney, probably initiates release of renin, stimulating the rennin-angiotensionaldosterone-system (RAAS). Antidiuretic hormone (ADH, also called vasopressin), is released from the hypothalamus and posterior pituitary during periods of stress. Both aldosterone and ADH attempt to conserve water and electrolytes to sustain a sufficient vascular volume.

  • Question 3

ORDER NOW FOR CUSTOMIZED SOLUTION PAPERS

Needs Grading

A 64-year-old Caucasian female presents to the clinic with vague symptoms of non- specific abdominal pain, myalgias, constipation, polyuria, and says she feels “fuzzy headed” much of the time. She had about of kidney stones a few weeks ago and she fortunately was able to pass the small stones without requiring lithotripsy or other interventions. She was told by the urologist to follow up with her primary care provider after the kidney stones has resolved. 

The APRN examining the patient orders a Chem 7 which revealed a serum Ca++ of 13.1 mg/dl. The APN believes the patient has primary hyperparathyroidism and refers the patient to an endocrinologist who does a complete work up and concurs with the APRN’s diagnosis.

Question:

What is the role of parathyroid hormone in the development of primary hyperparathyroidism? 

Correct Answer:  

Primary hyperparathyroidism is the unregulated overproduction of parathyroid hormone (PTH) resulting in abnormal calcium balance. PTH secretion is increased and is not under the usual feedback control mechanisms. The Ca++ level in the blood increase because of increased resorption and GI absorption of calcium but fails to inhibit PTH secretion at normal levels of calcium because the feedback threshold for calcium is set at a higher level in the abnormal parathyroid tissues. Hypercalcemia and hypophosphatemia are the clinical hallmarks of hyperparathyroidism.

  • Question 4

Needs Grading

A 64-year-old Caucasian female presents to the clinic with vague symptoms of non- specific abdominal pain, myalgias, constipation, polyuria, and says she feels “fuzzy headed” much of the time. She had a fracture of her right metatarsal without trauma and currently is wearing a walking boot. She also had a bout of kidney stones a few weeks ago and she fortunately was able to pass the small stones without requiring lithotripsy or other interventions. She was told by the urologist to follow up with her primary care provider after the kidney stones has resolved. 

 

The APRN examining the patient orders a Chem 12 which revealed a serum Ca++ of 13.1 mg/dl. The APRN believes the patient has primary hyperparathyroidism and refers the patient to an endocrinologist who does a complete work up and concurs with the APRN’s diagnosis.

 

Question 1 of 2:

 

Explain the processes involved in the formation of renal stones in patients with hyperparathyroidism. 

Correct Answer:  

Increased renal filtration load of calcium leads to hypercalciuria. Hypercalcemia also affects proximal renal tubular functions, causing metabolic acidosis and production of abnormally alkaline urine. PTH hypersecretion enhances real phosphate excretion and results in hypophosphatemia and hyperphosphatemia. The combination of hypercalciuria, alkaline urine, and hyperphosphaturia leads to the formation of renal stones

  • Question 5

Needs Grading

A 64-year-old Caucasian female presents to the clinic with vague symptoms of non- specific abdominal pain, myalgias, constipation, polyuria, and says she feels “fuzzy headed” much of the time. She had a fracture of her right metatarsal without trauma and currently is wearing a walking boot. She also had a bout of kidney stones a few weeks ago and she fortunately was able to pass the small stones without requiring lithotripsy or other interventions. She was told by the urologist to follow up with her primary care provider after the kidney stones has resolved. 

The APRN examining the patient orders a Chem 12 which revealed a serum Ca++ of 13.1 mg/dl. The APRN believes the patient has primary hyperparathyroidism and refers the patient to an endocrinologist who does a complete work up and concurs with the APRN’s diagnosis. NURS 6501 Advanced Pathophysiology Module 4 Knowledge Check

Question 2 of 2:

Explain how a patient with hyperparathyroidism is at risk for bone fractures.  

Correct Answer:  

Excessive osteoclastic and osteocytic activity results in increased bone reabsorption that weakens the bone. Kyphosis of the dorsal spine and compression fractures of the vertebral bodies are also seen in patients with hyperparathyroidism.

  • Question 6

Needs Grading

A 64-year-old Caucasian female who is 4 weeks status post total parathyroidectomy with forearm gland insertion presents to the general surgeon for her post-operative checkup. She states that her mouth feels numb and she feels “tingly all over. The surgeon suspects the patient has hypoparathyroidism secondary to the parathyroidectomy with delayed vascularization of the implanted gland. She orders a Chem 20 to determine what electrolyte abnormalities may be present. The labs reveal a serum Ca++ of 7.1 mg/dl (normal 8.5 mg/dl-10.5 mg/dl) and phosphorous level of 5.6 mg/dl (normal 2.4-4.1 mg/dl). 

 

Question:

 

What serious consequences of hypoparathyroidism occur and why? 

Correct Answer:  

Hypoparathyroidism occurs when there is destruction of the parathyroid glands (autoimmune, surgical), abnormal parathyroid gland development, altered regulation of PTH production, or impaired PTH action. The acute manifestations of hypoparathyroidism (postsurgical hypoparathyroidism) are due to acute hypocalcemia.  The hallmark of acute hypocalcemia is tetany, which is a disorder of neuromuscular irritability. The symptoms of tetany may be mild (perioral numbness, paresthesia of the hands and feet, muscle cramps) or severe (carpopedal spasm, laryngospasm, and focal or generalized seizures, which must be distinguished from the generalized tonic muscle contractions that occur in severe tetany). The classic physical findings in patients with neuromuscular irritability due to latent tetany are Trousseau’s and Chvostek’s signs.

  • Question 7

Needs Grading

A 17-year-old boy is brought to the pediatrician’s office by his parents who are concerned about their son’s weight loss despite eating more, frequent urination, unquenchable thirst, and fatigue that is interfering with his school/work activities. He had been seemingly healthy until about 3 months ago when his parents started noticing these symptoms but put these symptoms down to his busy schedule including a part time job. He admits to sleeping more and tires very easily. He denies any other symptoms. 

 

PMH-noncontributory. No surgeries or major medical problems. Usual colds and ear infections as a child

 

Allergies-none know

 

Family history- maternal uncle with “some kind of sugar diabetes problem” but parents unclear on the exact disease process

 

Social-denies alcohol, tobacco or illicit drug use. Not sexually active. Junior at local high school and works in a fast food store after school and on weekends.

 

Labs in office: random glucose 220 mg/dl.

 

Based on his symptoms and the glucose level, the pediatrician makes a tentative diagnosis of Diabetes Mellitus type 1 and refers the boy and his parents to an endocrinologist for further work up and management plan.

 

Question 1 of 6:

 

The patient exhibited classic signs of Type 1 diabetes. Explain the pathophysiology of “polydipsia.”

Correct Answer:  

Because elevated blood glucose levels, water is osmotically attracted from body cells which results in intracellular dehydration and hypothalamic stimulation of thirst.

  • Question 8

Needs Grading

A 17-year-old boy is brought to the pediatrician’s office by his parents who are concerned about their son’s weight loss despite eating more, frequent urination, unquenchable thirst, and fatigue that is interfering with his school/work activities. He had been seemingly healthy until about 3 months ago when his parents started noticing these symptoms but put these symptoms down to his busy schedule including a part time job. He admits to sleeping more and tires very easily. He denies any other symptoms. 

 

PMH-noncontributory. No surgeries or major medical problems. Usual colds and ear infections as a child

 

Allergies-none know

 

Family history- maternal uncle with “some kind of sugar diabetes problem” but parents unclear on the exact disease process

 

Social-denies alcohol, tobacco or illicit drug use. Not sexually active. Junior at local high school and works in a fast food store after school and on weekends.

 

Labs in office: random glucose 220 mg/dl.

 

Based on his symptoms and the glucose level, the pediatrician makes a tentative diagnosis of Diabetes Mellitus type 1 and refers the boy and his parents to an endocrinologist for further work up and management plan.

Question 2 of 6:

 

The patient exhibited classic signs of Type 1 diabetes. Explain the pathophysiology of “polyuria.”

Correct Answer:  

Hyperglycemia acts as an osmotic diuretic. The amount of glucose filtered by the glomeruli of the kidneys exceeds the amount that can be reabsorbed by the renal tubules. Glycosuria results accompanied by large amounts of water lost in the urine.

  • Question 9

Needs Grading

A 17-year-old boy is brought to the pediatrician’s office by his parents who are concerned about their son’s weight loss despite eating more, frequent urination, unquenchable thirst, and fatigue that is interfering with his school/work activities. He had been seemingly healthy until about 3 months ago when his parents started noticing these symptoms but put these symptoms down to his busy schedule including a part time job. He admits to sleeping more and tires very easily. He denies any other symptoms. 

PMH-noncontributory. No surgeries or major medical problems. Usual colds and ear infections as a child

Allergies-none know

Family history- maternal uncle with “some kind of sugar diabetes problem” but parents unclear on the exact disease process

Social-denies alcohol, tobacco or illicit drug use. Not sexually active. Junior at local high school and works in a fast food store after school and on weekends.

Labs in office: random glucose 220 mg/dl.

Based on his symptoms and the glucose level, the pediatrician makes a tentative diagnosis of Diabetes Mellitus type 1 and refers the boy and his parents to an endocrinologist for further work up and management plan.

Question 3 of 6:

The patient exhibited classic signs of Type 1 diabetes. Explain the pathophysiology of “polyphagia.”

Correct Answer:  

Depletion of cellular stores of carbohydrates, fats, and proteins results in cellular starvation and a corresponding increase in hunger.

  • Question 10

Needs Grading

A 17-year-old boy is brought to the pediatrician’s office by his parents who are concerned about their son’s weight loss despite eating more, frequent urination, unquenchable thirst, and fatigue that is interfering with his school/work activities. He had been seemingly healthy until about 3 months ago when his parents started noticing these symptoms but put these symptoms down to his busy schedule including a part time job. He admits to sleeping more and tires very easily. He denies any other symptoms. 

 

PMH-noncontributory. No surgeries or major medical problems. Usual colds and ear infections as a child

 

Allergies-none know

 

Family history- maternal uncle with “some kind of sugar diabetes problem” but parents unclear on the exact disease process

 

Social-denies alcohol, tobacco or illicit drug use. Not sexually active. Junior at local high school and works in a fast food store after school and on weekends.

 

Labs in office: random glucose 220 mg/dl.

 

Based on his symptoms and the glucose level, the pediatrician makes a tentative diagnosis of Diabetes Mellitus type 1 and refers the boy and his parents to an endocrinologist for further work up and management plan.

Question 4 of 6:

The patient exhibited classic signs of Type 1 diabetes. Explain the pathophysiology of “weight loss.”

Correct Answer:  

Weight loss occurs because of fluid loss in osmotic diuresis and the loss of body tissue as fat and proteins are used for energy as a result of the effects of insulin deficiency.

  • Question 11

Needs Grading

A 17-year-old boy is brought to the pediatrician’s office by his parents who are concerned about their son’s weight loss despite eating more, frequent urination, unquenchable thirst, and fatigue that is interfering with his school/work activities. He had been seemingly healthy until about 3 months ago when his parents started noticing these symptoms but put these symptoms down to his busy schedule including a part time job. He admits to sleeping more and tires very easily. He denies any other symptoms. 

PMH-noncontributory. No surgeries or major medical problems. Usual colds and ear infections as a child

Allergies-none know

Family history- maternal uncle with “some kind of sugar diabetes problem” but parents unclear on the exact disease process

Social-denies alcohol, tobacco or illicit drug use. Not sexually active. Junior at local high school and works in a fast food store after school and on weekends.

Labs in office: random glucose 220 mg/dl.

Based on his symptoms and the glucose level, the pediatrician makes a tentative diagnosis of Diabetes Mellitus type 1 and refers the boy and his parents to an endocrinologist for further work up and management plan.

Question 5 of 6:

The patient exhibited classic signs of Type 1 diabetes. Explain the pathophysiology of “fatigue.”

Correct Answer:  

Metabolic change result in poor use of food products, contributing to lethargy and fatigue. Sleep loss from severe nocturia also contributes to fatigue.

  • Question 12

Needs Grading

A 17-year-old boy is brought to the pediatrician’s office by his parents who are concerned about their son’s weight loss despite eating more, frequent urination, unquenchable thirst, and fatigue that is interfering with his school/work activities. He had been seemingly healthy until about 3 months ago when his parents started noticing these symptoms but put these symptoms down to his busy schedule including a part time job. He admits to sleeping more and tires very easily. He denies any other symptoms. 

PMH-noncontributory. No surgeries or major medical problems. Usual colds and ear infections as a child

Allergies-none know

Family history- maternal uncle with “some kind of sugar diabetes problem” but parents unclear on the exact disease process

Social-denies alcohol, tobacco or illicit drug use. Not sexually active. Junior at local high school and works in a fast food store after school and on weekends.

Labs in office: random glucose 220 mg/dl.

Based on his symptoms and the glucose level, the pediatrician makes a tentative diagnosis of Diabetes Mellitus type 1 and refers the boy and his parents to an endocrinologist for further work up and management plan.

Question 6 of 6:

How do genetics and environmental factors contribute to the development of Type 1 diabetes?

Correct Answer:  

Islet cell autoantibodies (ICAs) were detected in serum from patients with autoimmune polyendocrine deficiency. They have subsequently been identified in 85 percent of patients with newly diagnosed type 1 diabetes and in prediabetic people.

Autoantigens form on insulin producing beta cells and circulate in the blood and lymphatics. This leads to processing and presentation of autoantigen by antigen presenting cells

There is activation of T helper 1 lymphocytes and T helper 2 lymphocytes There is activation of macrophages that release IL-1 and TNFα and activation of autoantigen specific T cytotoxic CD8 cells.

There is activation of B lymphocytes to produce islet cell autoantibodies and antiGAD65 antibodies. This cascade results in destruction of beta cells with decreased insulin production.

  • Question 13

Needs Grading

A 17-year-old boy recently diagnosed with Type I diabetes is brought to the pediatrician’s office by his parents with a chief complaint of “having the flu”. His symptoms began 2 days ago, and he has vomited several times and has not eaten very much. He can’t remember if he took his prescribed insulin for several days because he felt so sick. Random glucose in the office reveals glucose 560 mg/dl and the pediatrician made arrangements for the patient to be admitted to the hospitalist service with an endocrinology consult. 

 

BP 124/80mmHg; HR 122bpm; Respirations 32 breaths/min; Temp 97.2˚F; PaO297% on RA

 

Admission labs: Hgb 14.6 g/dl; Hct 58%

 

CMP- Na+ 122mmol/L; K+ 5.3mmol/L; Glucose 560mg/dl; BUN 52mg/dl; Creatinine 4.9mg/dl;

 

Cl- 95mmol/L; Ca++ 8.8mmol/L; AST (SGOT) 248U/L; ALT 198U/L; CK 34/35 IU/L; Cholesterol 198mg/dl;

 

Phosphorus 6.8mg/dl; Acetone Moderate; LDH38U/L; Alkaline Phosphatase 132U/L.

 

Arterial blood gas values were as follows: pH 7.09; Paco220mm Hg; Po2100mm Hg; Sao2 98% (room air)

 

HCO3-7.5mmol/L; anion gap 19.4

 

A diagnosis of diabetic ketoacidosis was made, and the patient was transferred to the Intensive Care Unit (ICU) for close monitoring.

 

Question:

 

The hormones involved in intermediary metabolism, exclusive of insulin, that can participate in the development of diabetic ketoacidosis (DKA) are epinephrine, glucagon, cortisol, growth hormone. Describe how they participate in the development of DKA.

Correct Answer:  

The most important concept is that DKA is caused by insulin deficiency and an increase in counterregulatory hormones that include glucagon, catecholamines, cortisol, and growth hormone. These counter regulatory hormones normally antagonize insulin buy increasing glucose production and decreasing tissue use of glucose. Insulin deficiency results in decreased glucose uptake, increased fat mobilization with release of fatty acids and accelerated gluconeogenesis, glycogenesis, and ketogenesis. In the absence of insulin, the release of free fatty acids from adipocytes increases production of ketone bodies by the mitochondria of the liver that exceeds peripheral use. Accumulation of ketone bodies causes a drop in pH and triggers the buffering system associated with metabolic acidosis. Hyperketonemia may result from impaired use of ketones by peripheral tissue, which permits strong organic acids to circulate freely. Bicarbonate buffering then does not occur and the individual develops a metabolic acidosis.

  • Question 14

Needs Grading

A 67-year-old African American male presents to the clinic with a chief complaint that he has to “go to the bathroom all the time and I feel really weak.” He states that this has been going on for about 3 days but couldn’t come to the clinic sooner as he went to the Wound Care clinic for a dressing change to his right great toe that has been chronically infected, and he now has osteomyelitis. Patient with known Type II diabetes with poor control. His last HgA1was 10.2 %. He says he can’t afford the insulin he was prescribed and only takes half of the oral agent he was prescribed. Random glucose in the office revealed glucose of 890 mg/dl. He was immediately referred to the ED by the APRN for evaluation of suspected hyperosmolar hyperglycemic non ketotic syndrome (HHNKS). Also called hyperglycemic hyperosmolar state (HHS). 

 

Question:

 

Explain the underlying processes that lead to HHNKS or HHS.

Correct Answer:  

HHNKS differs from DKA in the degree of insulin deficiency (more profound I DKA) and the elevation of glucose levels and degree of fluid deficiency which are more marked in HHKS.

The basic underlying mechanism of HHS is a relative reduction in effective circulating insulin with a concomitant rise in counterregulatory hormones. Unlike patients with DKA, most patients with HHS do not develop significant ketoacidosis. Insulin remains available in amounts sufficient to inhibit lipolysis and ketogenesis but insufficient to prevent hyperglycemia. Hyperosmolarity itself may also decrease lipolysis, limiting the amount of free fatty acids available for ketogenesis.

Patients with HHS have a very high serum glucose concentration, a near normal serum bicarbonate level and pH, a serum osmolarity that is usually greater than 320 mOsm/L (normal 275–295 mOsm/L), and either absent or low levels of ketones in both the urine and serum. Electrolyte imbalances are common, with severe potassium deficits that need to be corrected, usually over several days. Phosphorous and sodium replacement may also be needed.

  • Question 15

Needs Grading

A 32-year-old woman presented to the clinic complaining of weight gain, swelling in her legs and ankles and a puffy face. She also recently developed hypertension and diabetes type 2. She noted poor short-term memory, irritability, excess hair growth (women), red-ruddy face, extra fat around her neck, fatigue, poor concentration, and menstrual irregularity in addition to muscle weakness. Given her physical appearance and history, a tentative diagnosis of hypercortical function was made. Diagnostics included serum and urinary cortisol and serum adrenocorticotropic hormone (ACTH). MRI revealed a pituitary adenoma. NURS 6501 Advanced Pathophysiology Module 4 Knowledge Check 

 

Question:

 

How would you differentiate Cushing’s disease from Cushing’s syndrome? 

Correct Answer:  

Cushing’s syndrome is a rare disease that is caused by the over production of cortisol by the adrenal glands. This can be caused by a tumor of the adrenal glands, the lungs, or the pituitary gland. When the tumor produces too much ACTH, it causes over production of cortisol by the adrenal glands. If the source is the pituitary, it is called Cushing’s disease. Cushing’s disease occurs more often in women than men and more often occurs between the ages of 20 and 40.

ACTH dependent hypercortisolism, the excess ACTH stimulates excess production of cortisol and loss of the negative feedback control of ACTH secretion. People with Cushing’s syndrome do not have diurnal or circadian secretion patterns of ACTH and cortisol, and they do not increase ACTH and cortisol secretion in response to stressors.

  • Question 16

Needs Grading

A 47-year-old female is referred to the endocrinologist for evaluation of her chronically elevated blood pressure, hypokalemia, and hypervolemia. The patient’s hypertension has been refractory to the usual medications such as beta blockers, diuretics, and angiotensin-converting enzyme (ACE) inhibitors. After a full work up including serum and urinary electrolyte levels, aldosterone suppression test, plasma aldosterone to renin ratio, and MRI which revealed an autonomous adenoma, the endocrinologist diagnoses the patient with primary hyper-aldosteronism. 

 

Question:

 

What is the pathogenesis of primary hyper-aldosteronism? 

Correct Answer:  

Primary hyperaldosteronism can be caused by either hyperactivity in one adrenal gland (unilateral disease) or both (bilateral disease). Unilateral disease is usually caused by an aldosterone producing adenoma (benign tumor) and less commonly by adrenal cancer or hyperplasia (when the whole gland is hyperactive). Excessive autonomous secretion of aldosterone without its principle regulator, angiotension II, causes hypokalemia and induces insulin resistance; promotes inflammation, endothelial dysfunction, and cardiovascular remodeling (increased left ventricular wall and carotid intima thickness. It also affects adipose tissue differentiation and function. Therefore, primary hyperaldosteronism can influence the features of metabolic syndrome, including hypertension, obesity, dyslipidemia, insulin resistance and hyperglycemia.

  • Question 17

Needs Grading

A 47-year-old African American male presents to the clinic with chief complaints of polyuria, polydipsia, polyphagia, and weight loss. He also said that his vison occasionally blurs and that his feet sometimes feel numb.  He has increased hunger despite weight loss and admits to feeling unusually tired. He also complains of “swelling” and enlargement of his abdomen. 

 

Past Medical History (PMH) significant for HTN fairly well controlled with and ACE inhibitor; central obesity, and dyslipidemia treated with a statin, Review of systems negative except for chief complaint. Physical exam unremarkable except for decreased filament test both feet. Random glucose in office 290 mg/dl. The APRN diagnoses the patient with type II DM and prescribes oral medication to control the glucose level and also referred the patient to a dietician for dietary teaching.

 

Question:

 

What is the basic underlying pathophysiology of Type II DM? 

Correct Answer:  

There are very complex interactions that result in the development of Type II diabetes. The pathophysiology of type 2 diabetes mellitus is characterized by peripheral insulin resistance, impaired regulation of hepatic glucose production, and declining β-cell function, eventually leading toβ -cell failure. Type 2 diabetes mellitus consists of a constellation of dysfunctions characterized by hyperglycemia and resulting from the combination of resistance to insulin action, inadequate insulin secretion, and excessive or inappropriate glucagon secretion. It is often associated with obesity.

  • Question 18

Needs Grading

A 21-year-old male was involved in a motorcycle accident and sustained a closed head injury. He is waking up and interacting with his family and medical team. He complained of thirst that doesn’t seem to go away no matter how much water he drinks. The nurses note that he has had 3500 cc of pale-yellow urine in the last 24 hours. Urine was sent for osmolality which was reported as 122 mOsm/L. A diagnosis of probable neurogenic diabetes insipidus was made. 

 

Question:

 

What causes diabetes insipidus (DI)? 

Correct Answer:  

Diabetes insipidus (DI) is defined as the passage of large volumes (>3 L/24 hr) of dilute urine (< 300 mOsm/kg). There are 2 major forms: Central (neurogenic, pituitary, or neurohypophyseal) DI, characterized by decreased secretion of antidiuretic hormone (ADH; also referred to as arginine vasopressin [AVP]). Nephrogenic DI characterized by decreased ability to concentrate urine because of resistance to ADH action in the kidney.

AVP is the primary determinant of free water excretion in the body. Its main target is the kidney, where it acts by altering the water permeability of the cortical and medullary collecting tubules. Water is reabsorbed by osmotic equilibration with the hypertonic interstitium and returned to the systemic circulation.

  • Question 19

Needs Grading

A 43-year-old female patient presents to the clinic with complaints of nervousness, racing heartbeat, anxiety, increased perspiration, heat intolerance, hyperactivity and palpitations. She states she had had the symptoms for several months but attributed the symptoms to beginning to care for her elderly mother who has Alzheimer’s Disease. She has lost 15 pounds in the last 3 months without dieting. Her past medical history is significant for rheumatoid arthritis that she has had for the last 10 years well controlled with methotrexate and prednisone. Physical exam is remarkable for periorbital edema, warm silky feeling skin, and palpable thyroid nodules in both lobes of the thyroid. Pending laboratory diagnostics, the APRN diagnoses the patient as having hyperthyroidism, also called Graves’ Disease. 

 

Question:

 

Explain how the negative feedback loop controls thyroid levels.

Correct Answer:  

Hyperthyroidism is a set of disorders that involve excessive synthesis and secretion of thyroid hormones T3 and T4. This unregulated release causes a hypermetabolic state that can lead to a serious condition called thyrotoxicosis. The thyroid gland is regulated by thyroid stimulating hormone (TSH) from the pituitary gland, which, in turn, is regulated by the hypothalamus via a negative feedback loop. Calcitonin, a hormone that affects blood calcium levels, is also secreted by the thyroid gland. Hyperthyroidism is known as Graves’ Disease. Genetic factors interacting with the environment triggers play an important role in the pathogenesis. It is classified as an autoimmune disease and often goes along with other autoimmune diseases. It results  from a failure of the feedback system. Normally, the secretion of thyroid hormone is controlled by a complex feedback mechanism involving the interaction of stimulatory and inhibitory factors (see the image below). Thyrotropin-releasing hormone (TRH) from the hypothalamus stimulates the pituitary to release TSH. In Graves’ disease, a circulating autoantibody against the thyrotropin receptor provides continuous stimulation of the thyroid gland. This stimulatory immunoglobulin has been called long-acting thyroid stimulator (LATS), thyroid-stimulating immunoglobulin (TSI), thyroid-stimulating antibody (TSab), and TSH-receptor antibody (TRab). These antibodies stimulate the production and release of thyroid hormones and thyroglobulin.

  • Question 20

Needs Grading

A 43-year-old female patient with known Graves’ Disease presents to the clinic with complaints of nervousness, racing heartbeat, anxiety, increased perspiration, heat intolerance, hyperactivity and severe palpitations. She states she had been given a prescription for propylthiouracil, an antithyroid medication but she did not fill the prescription as she claims she lost it. She had been given the option of thyroidectomy which she declined. She also notes that she is having trouble with her vision and often has blurry eyes. She states that her eyes seem “to bug out of her face”. She has had recurrent outs of nausea and vomiting. She was recently hospitalized for pneumonia.  Physical exam is significant for obvious exophthalmos and pretibial myxedema. Vital signs are temp 101.2˚F, HR 138 and irregular, BP 160/60 mmHg. Respirations 24. Electrocardiogram revealed atrial fibrillation with rapid ventricular response. The APRN recognizes the patient is experiencing symptoms of thyrotoxic crisis, also called thyroid storm. The patient was immediately transported to a hospital for critical care management. 

 

Question:

 

How did the patient develop thyroid storm? What were the patient factors that lead to the development of thyroid storm? 

Correct Answer:  

Thyroid storm is a hypermetabolic state induced by excessive release of thyroid hormones (THs) in individuals with thyrotoxicosis. Individuals may be undiagnosed or undertreated when they develop the symptoms. The symptoms are caused by the sudden release and action of thyroxine (T4) and triiodothyronine (T3) that exceeds metabolic demands. Symptoms must be rapidly treated in order to prevent life threatening complications such as high output heart failure, hyperthermia, delirium and hypovolemia from excessive vomiting.

  • Question 21

Needs Grading

A 44-year-old woman presents to the clinic with complaints of extreme fatigue, weight gain, decreased appetite, cold intolerance, dry skin, hair loss, and sleepiness. She also admits that she often bursts into tears without any reason and has been exceptionally forgetful. Her vision is occasionally blurry, and she admits to being depressed without any social or occupational triggers. Past medical history noncontributory. Physical exam Temp 96.2˚F, pulse 62 and regular, BP 108/90, respirations. Dull facial expression with coarse facial features. Periorbital puffiness noted. Based on the clinical history and physical exam, and pending laboratory data, the ARNP diagnoses the patient with hypothyroidism. 

 

Question:

 

What causes hypothyroidism? 

Correct Answer:  

Patients with primary hypothyroidism have elevated TSH levels and decreased free hormone levels. Patients with elevated TSH levels (usually 4.5- 10.0 mIU/L) but normal free hormone levels or estimates are considered to have mild or subclinical hypothyroidism. The most common cause of hypothyroidism is autoimmune thyroiditis known as Hashimoto’s disease. Primary hypothyroidism is essentially the only disease that is characterized by sustained rises in TSH levels. As the TSH level increases early in the disease, conversion of T4 to T3 increases, maintaining T3 levels. In early hypothyroidism, TSH levels are elevated, T4 levels are normal to low, and T3 levels are normal.

  • Question 22

Needs Grading

A 44-year-old woman is brought to the clinic by her husband who says his wife has had some mental status changes over the past few days. The patient had been previously diagnosed with hypothyroidism and had been placed on thyroid replacement therapy but had been lost to follow-up due to moving to another city for the husband’s work approximately 4 months ago. The patient states she lost the prescription bottle during the move and didn’t bother to have the prescription filled since she was feeling better. Physical exam revealed non-pitting, boggy edema around her eyes, hands and feet as well as the supraclavicular area. The APRN recognizes this patient had severe myxedema and referred the patient to the hospital for medical management. 

 

Question:

 

What causes myxedema coma? 

Correct Answer:  

In a patient with underlying hypothyroidism, inciting factors responsible for developing myxedema coma are numerous and include infection, trauma, cold exposure, or medications such as sedatives and anesthetics. Although most individuals are not comatose, they are at risk for significant cardiovascular and pulmonary complications. Patients will exhibit severe hypothermia without shivering, hypoventilation, hypotension, hypoglycemia, lactic acidosis ad coma.

  • Question 23

Needs Grading

A 53-year-old woman presents to the primary care clinic with complaints of severe headaches, palpitations, high blood pressure and diaphoresis. She relates that these symptoms come in clusters and when she has these “spells”, she also experiences, tremor, nausea, weakness, anxiety, and a sense of doom and dread, epigastric pain, and flank pain. She had one of these spells when she was at the pharmacy and the pharmacist took her blood pressure which was recorded as 200/118. The pharmacist recommended that she immediately be evaluated for these symptoms. Past medical history significant for a family history of neurofibromatosis type 1 (NF1). Based on the presenting symptoms and family history of NF1, the APRN suspects the patient has a pheochromocytoma. Laboratory data and computerized tomography of the abdomen confirms the diagnosis. 

Question 1 of 2:

 

What is a pheochromocytoma and how does it cause the classic symptoms the patient presented with? 

Correct Answer:  

Pheochromocytoma is an endocrine tumor of the adrenal gland that continuously secretes catecholamines (epinephrine and norepinephrine) in an uncontrolled fashion. Hypertension results from increase peripheral vascular resistance and may be sustained or paroxysmal. Because of excessive catecholamine secretion, pheochromocytomas may precipitate life-threatening hypertension or cardiac arrhythmias. Hypermetabolism and sweating are related to chronic activation of sympathetic receptors in adipocytes, hepatocytes and other tissue. Glucose intolerance may happen because of catecholamine induced inhibition of insulin release by the pancreas. NURS 6501 Advanced Pathophysiology Module 4 Knowledge Check

Catecholamines produced by pheochromocytomas are metabolized within chromaffin cells. Norepinephrine is metabolized to normetanephrine and epinephrine is metabolized to metanephrine. Because this process occurs within the tumor, independently of catecholamine release, pheochromocytomas are best diagnosed by measurement of these metabolites rather than by measurement of the parent catecholamines.

  • Question 24

Needs Grading

A 53-year-old woman presents to the primary care clinic with complaints of severe headaches, palpitations, high blood pressure and diaphoresis. She relates that these symptoms come in clusters and when she has these “spells”, she also experiences, tremor, nausea, weakness, anxiety, and a sense of doom and dread, epigastric pain, and flank pain. She had one of these spells when she was at the pharmacy and the pharmacist took her blood pressure which was recorded as 200/118. The pharmacist recommended that she immediately be evaluated for these symptoms. Past medical history significant for a family history of neurofibromatosis type 1 (NF1). Based on the presenting symptoms and family history of NF1, the APRN suspects the patient has a pheochromocytoma. Laboratory data and computerized tomography of the abdomen confirms the diagnosis. 

Question 2 of 2:

What are the treatment goals for managing pheochromocytoma? 

Correct Answer:  

Preoperative blockade of hormonally functional pheochromocytoma and paraganglioma (PPGL) to prevent cardiovascular complications is recommended, along with preoperative medical treatment to normalize blood pressure and heart rate and a  high-sodium diet with fluid intake to prevent severe hypotension after removal of the tumor. Blood pressure, heart rate, and glucose levels should be monitored immediately after surgery.

Minimally invasive (e.g., laparoscopic) adrenalectomy should be performed for most adrenal pheochromocytomas, with open resection reserved for very large or invasive pheochromocytomas; open resection is suggested for paragangliomas, but laparoscopic resection is an option for smaller tumors; partial adrenalectomy is also an option for certain patients.

Surgical resection of the tumor is the treatment of choice for pheochromocytoma and usually results in cure of the hypertension. Careful preoperative management is required to control blood pressure, correct fluid volume, and prevent intraoperative hypertensive crises.